Two particles in magnetic field QM

In summary: The answer to this question is a special case of the time evolution operator.In summary, the conversation discusses the problem of finding the wave function and probability of particles with spin S1=1/2 and S2=1/2 in a state with S=0 at t=0. The first part of the conversation involves finding the wave function at t=0 in the S1z and S2z basis. The second part introduces a magnetic field and a Hamiltonian, and the conversation continues to discuss the time evolution of the particles and the probability of finding them in an S=1 state at time t. Different approaches and equations are mentioned, such as Schrödinger's equation and the time evolution operator.
  • #1
gasar8
63
0

Homework Statement


Two particles with spin [itex]S_1={1 \over 2}[/itex] and [itex]S_2={1 \over 2}[/itex] are at t=0 in a state with [itex]S=0[/itex].
a) Find wave function at t=0 in [itex]S_{1z},[/itex], [itex]S_{2z}[/itex] basis.
b) Second particle is in a magnetic field [itex]B = (\sin\theta,0,\cos\theta)[/itex], the Hamiltonian is [itex]H=\lambda \vec{S_2} \cdot \vec{B}.[/itex] Find the probability that we find particles at time t in [itex]S=1[/itex] state.

The Attempt at a Solution


a) [itex] |10\rangle ={1 \over \sqrt{2}} \big(|{1 \over 2}-{1 \over 2}\rangle + |-{1 \over 2}{1 \over 2}\rangle\big)[/itex]

b) If I write Hamiltonian with Pauli matrices, I get:
[tex]\lambda \vec{S_2} \cdot \vec{B}=\frac{\lambda \hbar B}{2}
\left(
\begin{array}{cc}
\cos\theta & \sin\theta\\
\sin\theta & -\cos\theta
\end{array}
\right)
[/tex]

Then I wrote Schrödingers equation and got eigenvalues [itex]E=\pm \frac{\lambda \hbar B}{2}[/itex] and eigenvectors ([itex]\cot\theta \pm {1 \over \sin\theta},1)[/itex].
I should be right to this point, right? But now, I have some problems. Is the wave function really: [tex]|\psi,0\rangle= |\uparrow\rangle+ \bigg(\cot\theta \pm {1 \over \sin\theta} \bigg) |\downarrow\rangle.[/tex]
How do I normalize it? And after that, how do I find its time evolution? I tried to put Hamiltonian on both states, so [itex]H|\uparrow\rangle[/itex] and [itex]H|\downarrow\rangle,[/itex] but is it right? Because H is a matrix and [itex]|\uparrow\rangle=|{1 \over 2}{1 \over 2}\rangle[/itex] is vector, so I get another vector?
 
Physics news on Phys.org
  • #2
gasar8 said:
a) |10⟩=1√2(|12−12⟩+|−1212⟩)|10⟩=12(|12−12⟩+|−1212⟩) |10\rangle ={1 \over \sqrt{2}} \big(|{1 \over 2}-{1 \over 2}\rangle + |-{1 \over 2}{1 \over 2}\rangle\big)
No, that state corresponds to one of ##S=1## states.
gasar8 said:
Second particle is in a magnetic field B=(sinθ,0,cosθ)B=(sin⁡θ,0,cos⁡θ)B = (\sin\theta,0,\cos\theta), the Hamiltonian is H=λ→S2⋅⃗B.H=λS2→⋅B→.H=\lambda \vec{S_2} \cdot \vec{B}. Find the probability that we find particles at time t in S=1S=1S=1 state.
The evolution of the state is calculated by applying the operator ##e^{-iHt/\hbar}## to the initial state ##|00\rangle##, where that time evolution operator only acts on the second particle. So, first find the correct initial state.
 
  • #3
Ok, so the [itex]S=0[/itex] must be [itex]|00\rangle = {1 \over \sqrt{2}} \big(|{1 \over 2}-{1 \over 2}\rangle - |-{1 \over 2}{1 \over 2}\rangle\big)[/itex]

blue_leaf77 said:
Time evolution operator only acts on the second particle.
So I must apply Hamiltonian to [itex]|S_2 S_{z2}\rangle[/itex] like [itex]H |{1 \over 2}-{ 1 \over 2}\rangle[/itex] and [itex]H |{1 \over 2} {1 \over 2}\rangle[/itex]?
 
  • #4
gasar8 said:
So I must apply Hamiltonian to |S2Sz2⟩|S2Sz2⟩|S_2 S_{z2}\rangle like H|12−12⟩H|12−12⟩H |{1 \over 2}-{ 1 \over 2}\rangle and H|1212⟩H|1212⟩H |{1 \over 2} {1 \over 2}\rangle?
Not the Hamiltonian, instead the time evolution operator.
 
  • #5
So the time evolution operator is something like [itex]U=e^{-i{H \over \hbar}t}=e^{\pm i {\lambda B \over 2}t}.[/itex] But I am not sure how to apply it to an initial state, because there is also first particle in [itex]|00\rangle.[/itex] How do I know it applies only on the second? Is it just because there is a [itex]S_2[/itex] in Hamiltonian and I need to randomly choose one to be second?
So the time evolution must be:
[tex]|\psi(t)\rangle = c_{\uparrow} \ e^{- i {\lambda B \over 2}t} |\uparrow\rangle + c_{\downarrow} \ e^{ i {\lambda B \over 2}t}|\downarrow\rangle.[/tex]
Do I have to pick these two coefficients from [itex]|00\rangle= {1 \over \sqrt{2}} \big(|{1 \over 2}-{1 \over 2}\rangle - |-{1 \over 2}{1 \over 2}\rangle\big)[/itex], so the [itex]|S_2,S_{z2}\rangle = {1 \over \sqrt{2}} \big(|{1 \over 2}-{1 \over 2}\rangle - |{1 \over 2}{1 \over 2}\rangle \big).[/itex] Is this correct? So the wave function would look like:
[tex]|\psi(t)\rangle = - {1 \over \sqrt{2}} e^{- i {\lambda B \over 2}t} |\uparrow\rangle + {1 \over \sqrt{2}} e^{ i {\lambda B \over 2}t}|\downarrow\rangle.[/tex]

But where is the connection back to both particles and [itex]S=1[/itex] state? Btw, how do I even know which [itex]S=1[/itex] state do we need?
 
  • #6
gasar8 said:
So the time evolution operator is something like U=e−iHℏt=e±iλB2t.U=e−iHℏt=e±iλB2t.
No, you cannot write the last equality. The most right expression is just the eigenvalue of the middle one while the middle one is an operator. There is no way a scalar can be equal to an operator.
gasar8 said:
How do I know it applies only on the second? Is it just because there is a S2S2S_2 in Hamiltonian and I need to randomly choose one to be second?
It's because we know that the Hamiltonian ##H = \lambda \mathbf B \cdot \mathbf S_2## applies to one particle only. Only particle two is placed in a magnetic field, the other particle is not associated with any Hamiltonian. It might be strange that a particle does not have Hamiltonian and hence its time evolution is static but that's the consequence of neglecting the spatial wavefunction and not having this particle interact with magnetic field.
For the purpose of this problem, it's more convenient to separate the kets of the two particle. Thus
$$
|\uparrow \uparrow\rangle \rightarrow |\uparrow\rangle |\uparrow\rangle
$$
If I were to write the complete time evolution operator, it will be ##I\otimes e^{-iHt/\hbar}## where ##I## only acts on the first particle (the first particle is static in time). For example if you apply to ##|11\rangle##, it will be ##(I\otimes e^{-iHt/\hbar})|11\rangle = (I\otimes e^{-iHt/\hbar})(|\uparrow\rangle |\uparrow\rangle) = (I|\uparrow\rangle )(e^{-i\lambda \mathbf B \cdot \mathbf S_2t/\hbar} |\uparrow\rangle)##.
Now, in order to calculate ##e^{-i\lambda \mathbf B \cdot \mathbf S_2t/\hbar} |\uparrow\rangle##, I suggest that you expand the time evolution operator into Taylor series and simplify it using the properties of Pauli matrices. For this matter, you might be interested in the question part of this http://physics.stackexchange.com/questions/41697/what-is-the-spin-rotation-operator-for-spin-1-2.
 

1. What is the Schrödinger equation for two particles in a magnetic field?

The Schrödinger equation for two particles in a magnetic field is a time-dependent partial differential equation that describes the evolution of the wave function for two particles in the presence of a magnetic field. It is given by: iℏ ∂ψ/∂t = Hψ, where ψ is the wave function, t is time, is the reduced Planck's constant, and H is the Hamiltonian operator.

2. How does the magnetic field affect the energy levels of the two particles?

The magnetic field causes a splitting of the energy levels of the two particles. This is known as the Zeeman effect. The energy levels of the particles are shifted depending on their spin and the strength of the magnetic field. In the presence of a strong magnetic field, the energy levels become discrete and can be described by the Landau levels.

3. What is the physical significance of the quantum mechanical phase in two particle systems?

The quantum mechanical phase in two particle systems is significant as it describes the interference between the two particles. This interference can lead to phenomena such as entanglement, where the state of one particle cannot be described independently of the other. The phase also plays a crucial role in determining the probabilities for the particles to be in certain positions or states.

4. How does the magnetic field affect the motion of the two particles?

The magnetic field affects the motion of the two particles by exerting a force on them. This force is known as the Lorentz force and is given by F = q(v x B), where q is the charge of the particle, v is its velocity, and B is the magnetic field. The Lorentz force causes the particles to move in circular or helical paths depending on their initial velocity and the direction of the magnetic field.

5. How are the two particles described in a magnetic field using quantum mechanics?

The two particles in a magnetic field are described using the principles of quantum mechanics, which involve the use of wave functions and operators to describe their behavior. The particles are described by their spin, position, and momentum, which are all represented by operators. The wave function of the particles evolves over time according to the Schrödinger equation, taking into account the effects of the magnetic field on their energy levels and motion.

Similar threads

  • Advanced Physics Homework Help
Replies
4
Views
715
  • Advanced Physics Homework Help
Replies
1
Views
763
  • Advanced Physics Homework Help
Replies
17
Views
1K
  • Advanced Physics Homework Help
Replies
1
Views
1K
Replies
16
Views
549
  • Advanced Physics Homework Help
Replies
1
Views
922
  • Advanced Physics Homework Help
Replies
13
Views
1K
  • Advanced Physics Homework Help
Replies
3
Views
970
  • Advanced Physics Homework Help
Replies
9
Views
930
  • Advanced Physics Homework Help
Replies
0
Views
554
Back
Top